5
$\begingroup$

Let $G$ be the reductive group $\operatorname{GSp}_{4}$. Let $\pi$ be a smooth admissible cuspidal representation of $\operatorname{GSp}_{4}(\mathbb{A}^{(\infty)})$ of dominant weight. Assume, for caution, that $\pi$ satisfies a multiplicity one hypothesis.

Fix $p$ an odd prime. To $\pi$ is attached a $p$-adic representation $\rho$ of the absolute Galois group of $\mathbb{Q}$ unramified outside a finite set of finite places and such that the characteristic polynomial of the Frobenius morphisms $Fr\ell$ for $\ell$ outside this set coincides with the Euler factor at $\ell$ of the degree 4 $L$-function of $\pi$. This Galois representation occurs in the degree 3 cohomology of the étale cohomology of a Siegel-Shimura variety.

The image of complex conjugation under $\rho$ is semi-simple so can be chosen to be diagonal with eigenvalues 1 and -1. How many $-1$ are there?

$\endgroup$
1
  • $\begingroup$ I once went to a talk by Tilouine where he discussed some similar questions. You might want to look up some of his work. $\endgroup$
    – naf
    Apr 15, 2010 at 12:38

1 Answer 1

5
$\begingroup$

The preprint "Conjecture de type de Serre et formes compagnons pour $GSp_4$" by Florian Herzig and Tilouine (available here) indicates that there should be two 1's and two -1's.

EDIT: Having come in to work today, I could look up the more precise statement given in chapter 9 of Tilouine's book "Deformations of Galois representations and Hecke algebras" (incorrectly referenced in the aformentioned article of Herzig–Tilouine). The statement is the following: Let $G=GSp(4)$, let $\pi$ be a regular algebraic cuspidal automorphic representation of $G(\mathbf{A}_F)$, where $\mathbf{A}_F$ is the adele ring of a number field $F$. Let $\bar{\rho}$ be the associated mod $p$ Galois representation. Let $v$ be a real place of $F$. Then the $G$-conjugacy class of ${\bar\rho}(c_v)$ (where $c_v$ is a complex conjugation at $v$) contains the matrix $\text{diag}(1,1,-1,-1)$.

$\endgroup$
1
  • $\begingroup$ Thanks a lot! I will wait a bit more see if someone knows something unconditional (and do some research myself). If nothing new comes out of it, I'll accept your answer. $\endgroup$
    – Olivier
    Apr 16, 2010 at 8:48

Your Answer

By clicking “Post Your Answer”, you agree to our terms of service and acknowledge you have read our privacy policy.

Not the answer you're looking for? Browse other questions tagged or ask your own question.